Inscription / Connexion Nouveau Sujet
Niveau école ingénieur
Partager :

estimation

Posté par
toureissa
05-02-23 à 08:35

modération > **Bonjour***

On considère un échantillon (X1, ..Xn) issu de la loi normale N(N(\theta ,1) avec \theta \in \R
inconnu.
1. Montrer que \bar{X_n}=\frac{1}{N}\sum_{i=1}^{n}{X_i} est un estimateur efficace de \theta.

On suppose que \theta=0.
2. Soit U_n=\bar{X_n}1_{\{\lvert \bar{X_n} x \rvert \geq n^{-\frac{1}{4}}\}}. Quel est le biais de U_n
?

Posté par
toureissa
re : estimation 05-02-23 à 08:41

Bonjour à tous ,
Désolé , je voulais prévisualiser avant de poster, mais je n'ai pas preter attention.
Mon problème se trouve à la deuxième question. Je ne parvient pas a calculer l'Esperance de Un. J'ai essayer de partir de la formule de l covariance mais je n'y parvient pas.

Posté par
carpediem
re : estimation 05-02-23 à 10:01

salut

2/ c'est quoi ce x dans l'ensemble de l'indicatrice ?

tu veux un biais ... mais en estimant quoi ?

Posté par
toureissa
re : estimation 05-02-23 à 11:51

C'est une erreur de tape

Posté par
toureissa
re : estimation 05-02-23 à 12:47

surement estimer le paramètre du modelé.

Posté par
toureissa
re : estimation 05-02-23 à 14:11

J'ai pu résous le problème juste en calculant d'abord la densité de Xn. Merci



Vous devez être membre accéder à ce service...

Pas encore inscrit ?

1 compte par personne, multi-compte interdit !

Ou identifiez-vous :


Rester sur la page

Inscription gratuite

Fiches en rapport

parmi 1675 fiches de maths

Désolé, votre version d'Internet Explorer est plus que périmée ! Merci de le mettre à jour ou de télécharger Firefox ou Google Chrome pour utiliser le site. Votre ordinateur vous remerciera !